¿Cómo es compatible la mecánica cuántica con el límite de la velocidad de la luz?

Considere un electrón libre en el espacio. Supongamos que medimos su posición para estar en el punto A con un alto grado de precisión en el tiempo 0. Si recuerdo mi QM correctamente, a medida que pasa el tiempo, la función de onda se expande, y hay una posibilidad pequeña pero finita de encontrarlo bastante mucho en cualquier parte del universo. Supongamos que es medido un segundo después por un observador diferente a más de un segundo luz de distancia y, aunque es extremadamente improbable, este observador descubre ese electrón. Es decir, el electrón parece haber recorrido la distancia intermedia más rápido que la velocidad de la luz. ¿Que está pasando aqui?

Se me ocurren varias posibilidades, no necesariamente contradictorias:

  1. No recuerdo bien cómo funcionan las funciones de onda y, en particular, la función de onda tiene una amplitud cero (no solo muy pequeña) más allá del cono de velocidad de la luz.
  2. Dado que no podemos controlar este viaje, no se transmite información y, por lo tanto, se conserva la relatividad especial (similar a cómo las correlaciones no locales de los experimentos de tipo EPR no transmiten información)
  3. Aunque la diferencia entre las posiciones es mayor de lo que podría haber sido atravesada por el electrón que viaja en c, si hubiéramos medido el momento en su lugar, siempre habríamos encontrado que es menor que metro mi C y es realmente el impulso instantáneo lo que restringe la relatividad especial; no la distancia dividida por el tiempo.
  4. Mi pregunta está mal planteada, y de alguna manera sin sentido.

¿A alguien le importaría explicar cómo se resuelve este problema?

Esta es una de las razones por las que necesitamos la teoría cuántica de campos.
Bienvenida Elliotte, buena pregunta. No sé la respuesta, espero que alguien con un mejor conocimiento en QM pueda ayudarlo. Tengo una pequeña corrección para ti sobre el impulso. En relatividad especial, el impulso es pags = γ metro v , donde m es la masa en reposo, γ = 1 1 ( v C ) 2 y v es la velocidad. Como v tiende a c, γ tiende a infinito, por lo que el impulso puede ser mucho mayor que metro C .
Relacionado: physics.stackexchange.com/q/15282/2451 y enlaces allí.

Respuestas (3)

Excelente pregunta. Tiene razón sobre la propagación del paquete de ondas y, de hecho, obtiene una propagación superlumínica en QM no relativista, lo cual es basura. Necesitas una teoría relativista.

Debería leer la primera parte de las notas de la conferencia de Sidney Coleman sobre la teoría cuántica de campos, donde analiza este problema exacto: http://arxiv.org/abs/1110.5013

La respuesta corta es que necesita antipartículas. No hay forma de diferenciar entre un electrón que se propaga de A a B, con A a B separados como en el espacio, y un positrón que se propaga de B a A. Cuando agrega la amplitud para el último proceso, los efectos de la transmisión superlumínica se cancelan.

La forma de garantizar que todo funcione correctamente es acudir a una teoría cuántica relativista de campos. Estas teorías se construyen explícitamente de modo que todos los observables en una separación similar al espacio se conmutan entre sí, por lo que ninguna medición en A podría afectar las cosas en B si A y B son similares al espacio. Esta condición de causalidad restringe severamente el tipo de objetos que pueden aparecer en la teoría. Es la razón por la cual cada partícula necesita una antipartícula con la misma masa, espín y carga opuesta, y es parcialmente responsable del teorema de la estadística de espín (las partículas de espín entero son bosones y las partículas de espín medio entero son fermiones) y el teorema CPT ( la operación combinada de inversión de carga, reflexión especular e inversión de tiempo es una simetría exacta de la naturaleza).

Entonces, ¿es correcto decir que si uno libera un electrón en r=0 en t=0 y espera, entonces la probabilidad de medir el electrón fuera del cono de luz será cero, pero esto se debe al campo de un positrón? ¿Cuál cancela la propagación de electrones fuera del cono de luz? ¿Se puede entonces medir un positrón en cualquier lugar dentro del cono de luz?
Otro comentario es que no se necesita la teoría cuántica relativista de campos para este problema. La teoría de Dirac describe una partícula-electrón en propagación (no campo) lo suficientemente bien.
¡Gracias por la pregunta, Elliott! En mi clase de QFT, abordamos brevemente cómo el campo de antipartículas cancela los efectos superlumínicos del campo de partículas. Pero lo que no entiendo es que la partícula todavía puede viajar más rápido que la velocidad de la luz. ¿No hay manera de que uno pueda observar solo eso? Lo siento si esta es una pregunta tonta, solo he tomado un semestre de QFT... ¡Gracias!
¡Buenas preguntas, y nada tontas! @Alexey Bobrick: Tiene razón, no hay amplitud para medir un electrón fuera del cono de luz. Tampoco hay amplitud para medir un positrón dentro del cono de luz (¡si comienza con un estado de electrón en lugar de un estado de positrón!).
@ user34801: Su pregunta y la otra pregunta de Alexey son respondidas por la misma discusión: el "campo de electrones" ψ realmente es la suma de dos términos: un término que aniquila un electrón (la convención es al revés, culpan a Heisenberg) y un término que crea un positrón. El campo conjugado ψ ¯ hace al revés. (La acción opuesta en los estados de electrones frente a positrones lo convierte en un operador con carga eléctrica definida). Cualquier operador que actúe en estados de electrones o positrones debe construirse a partir de estas combinaciones para preservar la causalidad. Esta es la restricción que mencioné antes.
Cualquier intento de romper los operadores de campo en sus piezas no es físico y rompe la causalidad relativista de la teoría (aunque podría ser kosher descuidar las piezas de positrones en una aproximación no relativista de baja energía donde no hay positrones alrededor) . Lo que haces cuando mides la propagación de electrones es primero crear un electrón en x, luego aniquilarlo en otro lugar en y. La amplitud para esto es un elemento de matriz del operador ψ ( y ) ψ ¯ ( X ) . Un operador similar describe la propagación de positrones.
Cuando resuelves todo esto, descubres que las partículas solo se propagan dentro de su cono de luz. En ambos casos, dado que la carga se conserva, mides una partícula con la misma carga que la que creaste. La dirección del flujo de carga siempre es inequívoca. Si la memoria no me falla, el primer volumen de los textos QFT de Weinberg analiza todo esto, pero en su mayor parte es material estándar. No se preocupe si no lo aprendió en un semestre de QFT... ¡hay mucho que absorber en esta materia!

Comenta la respuesta de @Michael:

La respuesta corta es que necesitas antipartículas.

Es falso. En la Teoría Cuántica de Campos tiene soluciones que funcionan perfectamente también sin antipartículas, es decir, para campos reales. Incluso si desea considerar antipartículas, tenga siempre en cuenta que, a pesar del nombre engañoso , en realidad son partículas diferentes de las originales y decir que un electrón que se propaga de A a B es equivalente a un positrón que se propaga de B a A también es incorrecto: de hecho, hay una manera de distinguir entre los dos, a saber, el primero está representado por el campo y el segundo por su conjugado hermitiano y se transforman de manera diferente bajo la representación del grupo de Poincaré. Además, la suma de las dos contribuciones no anula los posibles factores superluminares.

Para responder a la pregunta original: QM de hecho no es una teoría relativista, fin de la historia. La extensión relativista correcta es QFT por el hecho de que las cancelaciones ocurren si se tienen en cuenta los grados de libertad que lleva el propio campo sobre los de las partículas (no es necesario tener antipartículas).

En un campo real, una partícula es su propia antipartícula. Todavía tiene un conmutador cero para mediciones separadas similares al espacio, debido a que la partícula que se propaga hacia adelante y hacia atrás se cancela. Y me encantaría ver por qué las dos contribuciones no cancelan los factores superlumínicos (recuerde que el único efecto superluminal observable sería un conmutador distinto de cero de campos separados similares al espacio)
La interpretación de que un campo real es su propia antipartícula sigue siendo al menos engañosa (si no errónea). Del mismo modo, las (anti)-partículas nunca viajan hacia atrás: siempre lo hacen hacia adelante y la terminología hacia atrás es solo para justificar (erróneamente) el signo menos. Sobre el conmutador: las dos contribuciones solo se anulan por campos libres; si intenta calcular el mismo conmutador para cualquier tipo de interacción, verá que las contribuciones generalmente no se cancelan entre sí (a menos que lo postule y derive los campos en consecuencia, pero esa es una historia diferente).

Las muy útiles soluciones de la ecuación de Shrodinger que generalmente se enseñan al principio de la mecánica cuántica no son invariantes de Lorenz y, por lo tanto, se pueden construir paradojas con respecto a la relatividad especial.

Las ecuaciones relativistas de Dirac:

La ecuación de Dirac es una ecuación de onda relativista derivada por el físico británico Paul Dirac en 1928. En su forma libre, o incluyendo interacciones electromagnéticas, describe todas las partículas de espín ½ masivo, para las cuales la paridad es una simetría, como los electrones y los quarks, y es consistente tanto con los principios de la mecánica cuántica como con la teoría de la relatividad especial,

La ecuación de Klein Gordon:

(a veces ecuación de Klein-Gordon- Fock) es una versión relativista de la ecuación de Schrödinger.

Por lo tanto, no hay problema con las soluciones simples de las funciones de onda subyacentes que se necesitan para desarrollar las teorías cuánticas de campo discutidas en las otras respuestas. Esos son un nivel meta que utiliza las soluciones de las ecuaciones relativistas como base sobre la cual operan los operadores de creación y aniquilación QFT.

En lo que respecta a la invariancia de Lorenz, es suficiente que el espacio de Hilbert en el que operan los operadores QFT sea invariante de Lorenz para no tener ningún problema de cono de luz con ningún modelo.